Lagrangian subspaces of symplectic vector spaces

Click For Summary
In a symplectic vector space (V, ω), a linear subspace Y with dimension equal to half of V must be shown to be Lagrangian, meaning Y equals its symplectic complement Y^ω. The discussion reveals confusion regarding the implications of the dimensionality condition, suggesting that if Y is not equal to Y^ω, one can find a vector w in Y that is not in Y^ω, leading to contradictions in dimensionality. The attempt to demonstrate that Y is isotropic or co-isotropic is hindered by these complications. Ultimately, a counterexample is presented using R^4, where a two-dimensional subspace can be symplectic but not Lagrangian, indicating that the original statement may not hold universally. The conclusion drawn is that the problem, as stated, is unsolvable in general.
Kreizhn
Messages
714
Reaction score
1

Homework Statement


If (V,\omega) is a symplectic vector space and Y is a linear subspace with \dim Y = \frac12 \dim V show that Y is Lagrangian; that is, show that Y = Y^\omega where Y^\omega is the symplectic complement.

The Attempt at a Solution



This is driving me crazy since I don't think it should be that hard. If we try to go directly, the fact that \dim Y = \frac12 \dim V implies that \dim Y^\omega = \frac12 \dim V. At this point, it is actually sufficient to show that Y is either isotropic or co-isotropic, since dimensional arguments will give equality. However, I cannot see why this is the case.

On the other hand, if we assume that Y \neq Y^\omega then without loss of generality, we may assume there exists w \in Y \setminus Y^\omega (otherwise, relabel Y and Y^\omega). The space W = \operatorname{span}(w) is isotropic so W \subseteq W^\omega and since W \subseteq Y then Y^\omega \subseteq W^\omega. I would like to put this together somehow, but I'm having trouble doing it.
 
Physics news on Phys.org
Nevermind. I've concluded that the question as phrased was unsolvable. Indeed, consider \mathbb R^4 with the standard symplectic basis \{x_i,y_i\}. The subspace generated by x1 and y1 is two-dimensional, but is in fact symplectic rather than Lagrangian. Hence the statement cannot be true in general.
 
Question: A clock's minute hand has length 4 and its hour hand has length 3. What is the distance between the tips at the moment when it is increasing most rapidly?(Putnam Exam Question) Answer: Making assumption that both the hands moves at constant angular velocities, the answer is ## \sqrt{7} .## But don't you think this assumption is somewhat doubtful and wrong?

Similar threads

Replies
15
Views
2K
Replies
8
Views
2K
Replies
2
Views
2K
  • · Replies 10 ·
Replies
10
Views
2K
  • · Replies 8 ·
Replies
8
Views
2K
  • · Replies 7 ·
Replies
7
Views
2K
  • · Replies 1 ·
Replies
1
Views
2K
  • · Replies 4 ·
Replies
4
Views
2K
Replies
1
Views
2K
  • · Replies 3 ·
Replies
3
Views
3K